GMAT Prep Question?

This topic has expert replies
Master | Next Rank: 500 Posts
Posts: 446
Joined: Thu Jul 26, 2007 1:07 pm
Thanked: 6 times

GMAT Prep Question?

by dferm » Fri Jul 25, 2008 6:08 am
What is the remainder when the positive integer x is divided by 6?

(1) When x is divided by 2, the remainder is 1; and when x is divided by 3 the remainder is zero

(2) When x is divided by 12, the remainder is 3.

Can someone put some light to this question.

Thanks..

Legendary Member
Posts: 661
Joined: Tue Jul 08, 2008 12:58 pm
Location: France
Thanked: 48 times

by pepeprepa » Fri Jul 25, 2008 6:13 am
Each one is sufficient --> D (not C I confused letters)
I do not frame the problem I just take examples

With 1) are possible 3, 9, 15, 21 .... and their remainder is 3 when divided by 6

With 2) Divided by 12 remainder is 3.... we have 3, 15, 27 and their remainder is 3 when divided by 6
Last edited by pepeprepa on Fri Jul 25, 2008 6:36 am, edited 2 times in total.

Master | Next Rank: 500 Posts
Posts: 446
Joined: Thu Jul 26, 2007 1:07 pm
Thanked: 6 times

by dferm » Fri Jul 25, 2008 6:20 am
Your Incorrect....Please give me an explanation if you can....

Newbie | Next Rank: 10 Posts
Posts: 1
Joined: Mon Jan 15, 2007 11:23 am

by apnamit » Fri Jul 25, 2008 6:22 am
With option 1 : 3, 9, 15, 21 ...

I have doubt here - since 3 is smaller than 6 , I am not sure if it can be considered as - 6x0 + 3. Any inputs ?

I think option two is sufficient to answer the question :

Remainder is 3 when divided by 12 , so number can be --- 15, 27, 39, 51 ...

In all the scenarios if the no. is divided by 6, the remainder is 3.

So, B i think.
Last edited by apnamit on Fri Jul 25, 2008 6:27 am, edited 1 time in total.

Legendary Member
Posts: 661
Joined: Tue Jul 08, 2008 12:58 pm
Location: France
Thanked: 48 times

by pepeprepa » Fri Jul 25, 2008 6:25 am
bug
Last edited by pepeprepa on Fri Jul 25, 2008 6:35 am, edited 1 time in total.

Legendary Member
Posts: 661
Joined: Tue Jul 08, 2008 12:58 pm
Location: France
Thanked: 48 times

by pepeprepa » Fri Jul 25, 2008 6:31 am
3/6 gives you a remainder of 3 like 1/3 gives you a remainder of 1
I learned it a few days ago

Legendary Member
Posts: 661
Joined: Tue Jul 08, 2008 12:58 pm
Location: France
Thanked: 48 times

by pepeprepa » Fri Jul 25, 2008 6:34 am
Answer for both statements are sufficient is D not C (Don't already know the letters perfectly)
That is perhaps why you told me I am wrong.

Master | Next Rank: 500 Posts
Posts: 446
Joined: Thu Jul 26, 2007 1:07 pm
Thanked: 6 times

by dferm » Fri Jul 25, 2008 6:35 am
the answer is D not C....

C would say that together they are sufficient...but D is each one by itself is sufficient....

Master | Next Rank: 500 Posts
Posts: 446
Joined: Thu Jul 26, 2007 1:07 pm
Thanked: 6 times

by dferm » Fri Jul 25, 2008 6:39 am
how does 3/6 give you a remainder of 3....not quite sure of that ....can you please put in a little input for me please.... Thanks... and how does 1/2 give you a remainder of 1....

I AM TOTALLY CONFUSED>......

Master | Next Rank: 500 Posts
Posts: 446
Joined: Thu Jul 26, 2007 1:07 pm
Thanked: 6 times

by dferm » Fri Jul 25, 2008 6:41 am
OK I can see how 1/3 can yield a remainder of 1 but not 3/6.........


CAN ANYBODY JUST PUT THIS QUESTION INTO PERSPECTIVE...

Legendary Member
Posts: 661
Joined: Tue Jul 08, 2008 12:58 pm
Location: France
Thanked: 48 times

by pepeprepa » Fri Jul 25, 2008 6:43 am

Master | Next Rank: 500 Posts
Posts: 446
Joined: Thu Jul 26, 2007 1:07 pm
Thanked: 6 times

by dferm » Fri Jul 25, 2008 6:47 am
Wrong post.... that question has nothing to do with this question being asked... I am asking how 3/6 is remainder 3......

Since you know why can you explain it to me.......

Legendary Member
Posts: 661
Joined: Tue Jul 08, 2008 12:58 pm
Location: France
Thanked: 48 times

by pepeprepa » Fri Jul 25, 2008 6:52 am
Read it first, the link is good.
I quote some lines.


pepeprepa wrote:
I know that now: "Every n divided by k with n<k has a remainder of n"


That's almost true- it's definitely true if n is greater than or equal to zero, and less than k. If n can be negative, then the above is not true.

GMAT/MBA Expert

User avatar
GMAT Instructor
Posts: 2621
Joined: Mon Jun 02, 2008 3:17 am
Location: Montreal
Thanked: 1090 times
Followed by:355 members
GMAT Score:780

by Ian Stewart » Fri Jul 25, 2008 8:02 am
The question asked above is "What is the remainder when 3 is divided by 6?"

First, how do we find remainders? If I ask, what is the remainder when 13 is divided by 6, I think we can all agree the answer is 1. To find the remainder here, we find the nearest multiple of 6 that is smaller than 13 -- which is 12 -- and then work out how much larger 13 is: 13 -12 = 1. We do the same when we divide 3 by 6; the nearest multiple of 6 that is smaller than 3 is zero (because 6*0 = 0), and 3 - 0 = 3, so the remainder is 3.

Hope that clears things up a bit!
For online GMAT math tutoring, or to buy my higher-level Quant books and problem sets, contact me at ianstewartgmat at gmail.com

ianstewartgmat.com